- Mon Oct 16, 2017 1:09 pm
#40563
Complete Question Explanation
(The complete setup for this game can be found here: lsat/viewtopic.php?t=15423)
The correct answer choice is (C)
The question stem creates a Z Y sequence. When the first rule is added to this new condition, a V Z Y sequence results. From the fourth rule, Y must be among the first three slots, and so the only placement for the three variables in the sequence is that Y must perform in slot three, Z must perform in slot two, and V must perform in slot one:
The last three slots are filled by U, W, and X, with the additional restriction that W X from the second rule. As there is no restriction on W performing in slot four, answer choice (C) is the correct answer.
(The complete setup for this game can be found here: lsat/viewtopic.php?t=15423)
The correct answer choice is (C)
The question stem creates a Z Y sequence. When the first rule is added to this new condition, a V Z Y sequence results. From the fourth rule, Y must be among the first three slots, and so the only placement for the three variables in the sequence is that Y must perform in slot three, Z must perform in slot two, and V must perform in slot one:
The last three slots are filled by U, W, and X, with the additional restriction that W X from the second rule. As there is no restriction on W performing in slot four, answer choice (C) is the correct answer.
You do not have the required permissions to view the files attached to this post.